$Parightarrow exists y Qy vdash exists y (Parightarrow Qy)$ using natural deduction












1












$begingroup$


$Parightarrow exists y Qy vdash exists y (Parightarrow Qy)$



My friend asked me to prove this using natural deduction. He knows I studied logic but I know little about natural deduction since I studied logic only by hilbert style.



Using inference rules he presented I thought a lot, but couldn't find any proof easier than these two. First, introduce $Parightarrow exists y Qy$ as a premise.



1-1. Assume $negexists y(Parightarrow Qy)$ and derive $forall yneg(Parightarrow Qy)$



1-2. Derive $neg(Parightarrow Qb)$ and find subproof of $(Pawedge neg Qb)$



1-3. Derive $forall yneg Qy$ from $neg Qb$ and $exists y Qy$ from $Pa$



1-4. Derive $negexists yQy$ from $forall y neg Qy$ and conclude there's a contradiction



Here's another.



2-1. Drive $neg Pa veeexists yQy$



2-2. Derive $Parightarrow Qb$ each from $neg Pa$ and $exists y Qy$ and conclude $exists y(Parightarrow Qy)$



Since I haven't tried this in a full content, I don't know how long proof will be but I guess it will be.. I think there should be easier proof than these but I don't know how to find it.










share|cite|improve this question











$endgroup$








  • 2




    $begingroup$
    It might be worth noting that any proof will necessarily involve something like excluded middle or double negation elimination - the statement is not valid in intuitionistic first-order logic.
    $endgroup$
    – Daniel Schepler
    Dec 9 '18 at 0:28
















1












$begingroup$


$Parightarrow exists y Qy vdash exists y (Parightarrow Qy)$



My friend asked me to prove this using natural deduction. He knows I studied logic but I know little about natural deduction since I studied logic only by hilbert style.



Using inference rules he presented I thought a lot, but couldn't find any proof easier than these two. First, introduce $Parightarrow exists y Qy$ as a premise.



1-1. Assume $negexists y(Parightarrow Qy)$ and derive $forall yneg(Parightarrow Qy)$



1-2. Derive $neg(Parightarrow Qb)$ and find subproof of $(Pawedge neg Qb)$



1-3. Derive $forall yneg Qy$ from $neg Qb$ and $exists y Qy$ from $Pa$



1-4. Derive $negexists yQy$ from $forall y neg Qy$ and conclude there's a contradiction



Here's another.



2-1. Drive $neg Pa veeexists yQy$



2-2. Derive $Parightarrow Qb$ each from $neg Pa$ and $exists y Qy$ and conclude $exists y(Parightarrow Qy)$



Since I haven't tried this in a full content, I don't know how long proof will be but I guess it will be.. I think there should be easier proof than these but I don't know how to find it.










share|cite|improve this question











$endgroup$








  • 2




    $begingroup$
    It might be worth noting that any proof will necessarily involve something like excluded middle or double negation elimination - the statement is not valid in intuitionistic first-order logic.
    $endgroup$
    – Daniel Schepler
    Dec 9 '18 at 0:28














1












1








1


1



$begingroup$


$Parightarrow exists y Qy vdash exists y (Parightarrow Qy)$



My friend asked me to prove this using natural deduction. He knows I studied logic but I know little about natural deduction since I studied logic only by hilbert style.



Using inference rules he presented I thought a lot, but couldn't find any proof easier than these two. First, introduce $Parightarrow exists y Qy$ as a premise.



1-1. Assume $negexists y(Parightarrow Qy)$ and derive $forall yneg(Parightarrow Qy)$



1-2. Derive $neg(Parightarrow Qb)$ and find subproof of $(Pawedge neg Qb)$



1-3. Derive $forall yneg Qy$ from $neg Qb$ and $exists y Qy$ from $Pa$



1-4. Derive $negexists yQy$ from $forall y neg Qy$ and conclude there's a contradiction



Here's another.



2-1. Drive $neg Pa veeexists yQy$



2-2. Derive $Parightarrow Qb$ each from $neg Pa$ and $exists y Qy$ and conclude $exists y(Parightarrow Qy)$



Since I haven't tried this in a full content, I don't know how long proof will be but I guess it will be.. I think there should be easier proof than these but I don't know how to find it.










share|cite|improve this question











$endgroup$




$Parightarrow exists y Qy vdash exists y (Parightarrow Qy)$



My friend asked me to prove this using natural deduction. He knows I studied logic but I know little about natural deduction since I studied logic only by hilbert style.



Using inference rules he presented I thought a lot, but couldn't find any proof easier than these two. First, introduce $Parightarrow exists y Qy$ as a premise.



1-1. Assume $negexists y(Parightarrow Qy)$ and derive $forall yneg(Parightarrow Qy)$



1-2. Derive $neg(Parightarrow Qb)$ and find subproof of $(Pawedge neg Qb)$



1-3. Derive $forall yneg Qy$ from $neg Qb$ and $exists y Qy$ from $Pa$



1-4. Derive $negexists yQy$ from $forall y neg Qy$ and conclude there's a contradiction



Here's another.



2-1. Drive $neg Pa veeexists yQy$



2-2. Derive $Parightarrow Qb$ each from $neg Pa$ and $exists y Qy$ and conclude $exists y(Parightarrow Qy)$



Since I haven't tried this in a full content, I don't know how long proof will be but I guess it will be.. I think there should be easier proof than these but I don't know how to find it.







logic predicate-logic natural-deduction






share|cite|improve this question















share|cite|improve this question













share|cite|improve this question




share|cite|improve this question








edited Dec 9 '18 at 11:31









Mauro ALLEGRANZA

66.1k449114




66.1k449114










asked Dec 8 '18 at 16:29









fbgfbg

423211




423211








  • 2




    $begingroup$
    It might be worth noting that any proof will necessarily involve something like excluded middle or double negation elimination - the statement is not valid in intuitionistic first-order logic.
    $endgroup$
    – Daniel Schepler
    Dec 9 '18 at 0:28














  • 2




    $begingroup$
    It might be worth noting that any proof will necessarily involve something like excluded middle or double negation elimination - the statement is not valid in intuitionistic first-order logic.
    $endgroup$
    – Daniel Schepler
    Dec 9 '18 at 0:28








2




2




$begingroup$
It might be worth noting that any proof will necessarily involve something like excluded middle or double negation elimination - the statement is not valid in intuitionistic first-order logic.
$endgroup$
– Daniel Schepler
Dec 9 '18 at 0:28




$begingroup$
It might be worth noting that any proof will necessarily involve something like excluded middle or double negation elimination - the statement is not valid in intuitionistic first-order logic.
$endgroup$
– Daniel Schepler
Dec 9 '18 at 0:28










1 Answer
1






active

oldest

votes


















1












$begingroup$

The first proof works, also if you need a lot of sub-proof to "play with" the propositional and quantifiers equivalences.



A more straightforward proof will be :



1) $Pa → ∃yQy$ --- premise



2) $lnot Pa lor Pa$ --- Excluded Middle



3) $lnot Pa$ --- first sub-proof from 2) by $lor$-elim



4) $Pa$ --- assumed [a]



5) $bot$



6) $Qb$ --- from 5)



7) $Pa to Qb$ --- by $→$-intro, discharging [a]




8) $∃y(Pa to Qy)$ --- by $∃$-intro, closing 1st sub-proof




9) $Pa$ --- second sub-proof from 2) by $lor$-elim



10) $∃yQy$ --- from 1) and 9) by $→$-elim



11) $Qb$ --- assumed from 10) for $∃$-elim



12) $Pa$ --- reiteration of 9)



13) $Pa to Qb$ --- by $→$-intro, discharging 12)




14) $∃y(Pa to Qy)$ --- by $∃$-intro, closing the $∃$-elim sub-proof and closing 2nd sub-proof




15) $∃y(Pa → Qy)$ --- from 3)-8) and 9)-14) and 2) by $lor$-elim.








share|cite|improve this answer











$endgroup$









  • 1




    $begingroup$
    I also thought about this before. what I was worried about is second assumption [b] exists after assuming [a]. So I thought that [b] should handled first and then [a]. I couldn't find such a way so I give up this method before
    $endgroup$
    – fbg
    Dec 8 '18 at 17:21













Your Answer





StackExchange.ifUsing("editor", function () {
return StackExchange.using("mathjaxEditing", function () {
StackExchange.MarkdownEditor.creationCallbacks.add(function (editor, postfix) {
StackExchange.mathjaxEditing.prepareWmdForMathJax(editor, postfix, [["$", "$"], ["\\(","\\)"]]);
});
});
}, "mathjax-editing");

StackExchange.ready(function() {
var channelOptions = {
tags: "".split(" "),
id: "69"
};
initTagRenderer("".split(" "), "".split(" "), channelOptions);

StackExchange.using("externalEditor", function() {
// Have to fire editor after snippets, if snippets enabled
if (StackExchange.settings.snippets.snippetsEnabled) {
StackExchange.using("snippets", function() {
createEditor();
});
}
else {
createEditor();
}
});

function createEditor() {
StackExchange.prepareEditor({
heartbeatType: 'answer',
autoActivateHeartbeat: false,
convertImagesToLinks: true,
noModals: true,
showLowRepImageUploadWarning: true,
reputationToPostImages: 10,
bindNavPrevention: true,
postfix: "",
imageUploader: {
brandingHtml: "Powered by u003ca class="icon-imgur-white" href="https://imgur.com/"u003eu003c/au003e",
contentPolicyHtml: "User contributions licensed under u003ca href="https://creativecommons.org/licenses/by-sa/3.0/"u003ecc by-sa 3.0 with attribution requiredu003c/au003e u003ca href="https://stackoverflow.com/legal/content-policy"u003e(content policy)u003c/au003e",
allowUrls: true
},
noCode: true, onDemand: true,
discardSelector: ".discard-answer"
,immediatelyShowMarkdownHelp:true
});


}
});














draft saved

draft discarded


















StackExchange.ready(
function () {
StackExchange.openid.initPostLogin('.new-post-login', 'https%3a%2f%2fmath.stackexchange.com%2fquestions%2f3031314%2fpa-rightarrow-exists-y-qy-vdash-exists-y-pa-rightarrow-qy-using-natural-d%23new-answer', 'question_page');
}
);

Post as a guest















Required, but never shown

























1 Answer
1






active

oldest

votes








1 Answer
1






active

oldest

votes









active

oldest

votes






active

oldest

votes









1












$begingroup$

The first proof works, also if you need a lot of sub-proof to "play with" the propositional and quantifiers equivalences.



A more straightforward proof will be :



1) $Pa → ∃yQy$ --- premise



2) $lnot Pa lor Pa$ --- Excluded Middle



3) $lnot Pa$ --- first sub-proof from 2) by $lor$-elim



4) $Pa$ --- assumed [a]



5) $bot$



6) $Qb$ --- from 5)



7) $Pa to Qb$ --- by $→$-intro, discharging [a]




8) $∃y(Pa to Qy)$ --- by $∃$-intro, closing 1st sub-proof




9) $Pa$ --- second sub-proof from 2) by $lor$-elim



10) $∃yQy$ --- from 1) and 9) by $→$-elim



11) $Qb$ --- assumed from 10) for $∃$-elim



12) $Pa$ --- reiteration of 9)



13) $Pa to Qb$ --- by $→$-intro, discharging 12)




14) $∃y(Pa to Qy)$ --- by $∃$-intro, closing the $∃$-elim sub-proof and closing 2nd sub-proof




15) $∃y(Pa → Qy)$ --- from 3)-8) and 9)-14) and 2) by $lor$-elim.








share|cite|improve this answer











$endgroup$









  • 1




    $begingroup$
    I also thought about this before. what I was worried about is second assumption [b] exists after assuming [a]. So I thought that [b] should handled first and then [a]. I couldn't find such a way so I give up this method before
    $endgroup$
    – fbg
    Dec 8 '18 at 17:21


















1












$begingroup$

The first proof works, also if you need a lot of sub-proof to "play with" the propositional and quantifiers equivalences.



A more straightforward proof will be :



1) $Pa → ∃yQy$ --- premise



2) $lnot Pa lor Pa$ --- Excluded Middle



3) $lnot Pa$ --- first sub-proof from 2) by $lor$-elim



4) $Pa$ --- assumed [a]



5) $bot$



6) $Qb$ --- from 5)



7) $Pa to Qb$ --- by $→$-intro, discharging [a]




8) $∃y(Pa to Qy)$ --- by $∃$-intro, closing 1st sub-proof




9) $Pa$ --- second sub-proof from 2) by $lor$-elim



10) $∃yQy$ --- from 1) and 9) by $→$-elim



11) $Qb$ --- assumed from 10) for $∃$-elim



12) $Pa$ --- reiteration of 9)



13) $Pa to Qb$ --- by $→$-intro, discharging 12)




14) $∃y(Pa to Qy)$ --- by $∃$-intro, closing the $∃$-elim sub-proof and closing 2nd sub-proof




15) $∃y(Pa → Qy)$ --- from 3)-8) and 9)-14) and 2) by $lor$-elim.








share|cite|improve this answer











$endgroup$









  • 1




    $begingroup$
    I also thought about this before. what I was worried about is second assumption [b] exists after assuming [a]. So I thought that [b] should handled first and then [a]. I couldn't find such a way so I give up this method before
    $endgroup$
    – fbg
    Dec 8 '18 at 17:21
















1












1








1





$begingroup$

The first proof works, also if you need a lot of sub-proof to "play with" the propositional and quantifiers equivalences.



A more straightforward proof will be :



1) $Pa → ∃yQy$ --- premise



2) $lnot Pa lor Pa$ --- Excluded Middle



3) $lnot Pa$ --- first sub-proof from 2) by $lor$-elim



4) $Pa$ --- assumed [a]



5) $bot$



6) $Qb$ --- from 5)



7) $Pa to Qb$ --- by $→$-intro, discharging [a]




8) $∃y(Pa to Qy)$ --- by $∃$-intro, closing 1st sub-proof




9) $Pa$ --- second sub-proof from 2) by $lor$-elim



10) $∃yQy$ --- from 1) and 9) by $→$-elim



11) $Qb$ --- assumed from 10) for $∃$-elim



12) $Pa$ --- reiteration of 9)



13) $Pa to Qb$ --- by $→$-intro, discharging 12)




14) $∃y(Pa to Qy)$ --- by $∃$-intro, closing the $∃$-elim sub-proof and closing 2nd sub-proof




15) $∃y(Pa → Qy)$ --- from 3)-8) and 9)-14) and 2) by $lor$-elim.








share|cite|improve this answer











$endgroup$



The first proof works, also if you need a lot of sub-proof to "play with" the propositional and quantifiers equivalences.



A more straightforward proof will be :



1) $Pa → ∃yQy$ --- premise



2) $lnot Pa lor Pa$ --- Excluded Middle



3) $lnot Pa$ --- first sub-proof from 2) by $lor$-elim



4) $Pa$ --- assumed [a]



5) $bot$



6) $Qb$ --- from 5)



7) $Pa to Qb$ --- by $→$-intro, discharging [a]




8) $∃y(Pa to Qy)$ --- by $∃$-intro, closing 1st sub-proof




9) $Pa$ --- second sub-proof from 2) by $lor$-elim



10) $∃yQy$ --- from 1) and 9) by $→$-elim



11) $Qb$ --- assumed from 10) for $∃$-elim



12) $Pa$ --- reiteration of 9)



13) $Pa to Qb$ --- by $→$-intro, discharging 12)




14) $∃y(Pa to Qy)$ --- by $∃$-intro, closing the $∃$-elim sub-proof and closing 2nd sub-proof




15) $∃y(Pa → Qy)$ --- from 3)-8) and 9)-14) and 2) by $lor$-elim.









share|cite|improve this answer














share|cite|improve this answer



share|cite|improve this answer








edited Dec 8 '18 at 19:14

























answered Dec 8 '18 at 16:42









Mauro ALLEGRANZAMauro ALLEGRANZA

66.1k449114




66.1k449114








  • 1




    $begingroup$
    I also thought about this before. what I was worried about is second assumption [b] exists after assuming [a]. So I thought that [b] should handled first and then [a]. I couldn't find such a way so I give up this method before
    $endgroup$
    – fbg
    Dec 8 '18 at 17:21
















  • 1




    $begingroup$
    I also thought about this before. what I was worried about is second assumption [b] exists after assuming [a]. So I thought that [b] should handled first and then [a]. I couldn't find such a way so I give up this method before
    $endgroup$
    – fbg
    Dec 8 '18 at 17:21










1




1




$begingroup$
I also thought about this before. what I was worried about is second assumption [b] exists after assuming [a]. So I thought that [b] should handled first and then [a]. I couldn't find such a way so I give up this method before
$endgroup$
– fbg
Dec 8 '18 at 17:21






$begingroup$
I also thought about this before. what I was worried about is second assumption [b] exists after assuming [a]. So I thought that [b] should handled first and then [a]. I couldn't find such a way so I give up this method before
$endgroup$
– fbg
Dec 8 '18 at 17:21




















draft saved

draft discarded




















































Thanks for contributing an answer to Mathematics Stack Exchange!


  • Please be sure to answer the question. Provide details and share your research!

But avoid



  • Asking for help, clarification, or responding to other answers.

  • Making statements based on opinion; back them up with references or personal experience.


Use MathJax to format equations. MathJax reference.


To learn more, see our tips on writing great answers.




draft saved


draft discarded














StackExchange.ready(
function () {
StackExchange.openid.initPostLogin('.new-post-login', 'https%3a%2f%2fmath.stackexchange.com%2fquestions%2f3031314%2fpa-rightarrow-exists-y-qy-vdash-exists-y-pa-rightarrow-qy-using-natural-d%23new-answer', 'question_page');
}
);

Post as a guest















Required, but never shown





















































Required, but never shown














Required, but never shown












Required, but never shown







Required, but never shown

































Required, but never shown














Required, but never shown












Required, but never shown







Required, but never shown







Popular posts from this blog

Plaza Victoria

In PowerPoint, is there a keyboard shortcut for bulleted / numbered list?

How to put 3 figures in Latex with 2 figures side by side and 1 below these side by side images but in...